Infinite sum

Algebra Level 4

If the sum n = 2 1 8 n 3 2 n \sum_{n=2}^{\infty}\frac1{8n^3-2n} is equal to -a/b+cln(2),where a,b,c are positive integers and a/b is expressed in simplest form,find the value of 11a+13b+17c


The answer is 78.

This section requires Javascript.
You are seeing this because something didn't load right. We suggest you, (a) try refreshing the page, (b) enabling javascript if it is disabled on your browser and, finally, (c) loading the non-javascript version of this page . We're sorry about the hassle.

2 solutions

Mateus Gomes
Feb 9, 2016

n = 2 1 8 n 3 2 n = ( 1 2 ) n = 2 [ 1 n ( 2 n 1 ) ( 2 n + 1 ) ] = ( 1 4 ) n = 2 [ 1 n ( 2 n 1 ) 1 n ( 2 n + 1 ) ] = ( 1 4 ) n = 2 [ 2 ( 2 n 1 ) 1 n ] [ 1 n 2 ( 2 n + 1 ) ] = ( 1 4 ) n = 2 [ 2 ( 2 n 1 ) 2 n + 2 ( 2 n + 1 ) ] = ( 1 2 ) n = 2 [ 1 ( 2 n 1 ) 1 n + 1 ( 2 n + 1 ) ] \sum_{n=2}^{\infty}\frac1{8n^3-2n}=(\frac{1}{2})\sum_{n=2}^{\infty}[\frac{1}{n(2n-1)(2n+1)}]=(\frac{1}{4})\sum_{n=2}^{\infty}[\frac{1}{n(2n-1)}-\frac{1}{n(2n+1)}]=(\frac{1}{4})\sum_{n=2}^{\infty}[\frac{2}{(2n-1)}-\frac{1}{n}]-[\frac{1}{n}-\frac{2}{(2n+1)}]=(\frac{1}{4})\sum_{n=2}^{\infty}[\frac{2}{(2n-1)}-\frac{2}{n}+\frac{2}{(2n+1)}]=(\frac{1}{2})\sum_{n=2}^{\infty}[\frac{1}{(2n-1)}-\frac{1}{n}+\frac{1}{(2n+1)}] ( 1 2 ) [ n = 2 1 ( 2 n 1 ) n = 2 1 n + n = 2 1 ( 2 n + 1 ) ] (\frac{1}{2})[\sum_{n=2}^{\infty}\frac{1}{(2n-1)}-\sum_{n=2}^{\infty}\frac{1}{n}+\sum_{n=2}^{\infty}\frac{1}{(2n+1)}] ( 1 2 ) [ n = 1 1 ( 2 n + 1 ) n = 2 1 n + n = 2 1 ( 2 n + 1 ) ] (\frac{1}{2})[\sum_{n=1}^{\infty}\frac{1}{(2n+1)}-\sum_{n=2}^{\infty}\frac{1}{n}+\sum_{n=2}^{\infty}\frac{1}{(2n+1)}] ( 1 2 ) [ 1 3 + 2 n = 2 1 ( 2 n + 1 ) n = 2 1 n ] (\frac{1}{2})[\frac{1}{3}+2\sum_{n=2}^{\infty}\frac{1}{(2n+1)}-\sum_{n=2}^{\infty}\frac{1}{n}] ( 1 2 ) [ 1 3 + 2 ( n = 2 1 ( 2 n + 1 ) 1 2 n = 2 1 n ) ] (\frac{1}{2})[\frac{1}{3}+2(\sum_{n=2}^{\infty}\frac{1}{(2n+1)}-\frac{1}{2}\sum_{n=2}^{\infty}\frac{1}{n})] 1 1 2 + 1 3 1 4 + 1 5 1 6 + . . . = n = 1 ( 1 ) n 1 n = l o g e 2 \Large\color{#3D99F6}{\boxed{\color{forestgreen}{\boxed{1-\frac{1}{2}+\frac{1}{3}-\frac{1}{4}+\frac{1}{5}-\frac{1}{6}+...=\sum_{n=1}^{\infty}\frac{({-1})^{n-1}}{n}=log_{e} 2}}}} ( 1 2 ) [ 1 3 + 2 ( n = 2 1 ( 2 n + 1 ) 1 2 n = 2 1 n ) = ( 1 2 ) [ 1 3 + 2 ( l o g e 2 5 6 ) ] (\frac{1}{2})[\frac{1}{3}+2(\sum_{n=2}^{\infty}\frac{1}{(2n+1)}-\frac{1}{2}\sum_{n=2}^{\infty}\frac{1}{n})=(\frac{1}{2})[\frac{1}{3}+2(log_{e} 2-\frac{5}{6})] ( 1 2 ) [ 1 3 + ( 2 l o g e 2 5 3 ) ] (\frac{1}{2})[\frac{1}{3}+(2log_{e} 2-\frac{5}{3})] ( 1 2 ) [ ( 2 l o g e 2 4 3 ) ] (\frac{1}{2})[(2log_{e} 2-\frac{4}{3})] ( l o g e 2 2 3 ) \rightarrow\Large\color{#3D99F6}{\boxed{(log_{e} 2-\frac{2}{3})}}

Krishna Sharma
Oct 18, 2014

It should be

2 3 + l n ( 2 ) \displaystyle -\frac{2}{3} + ln(2)

yes.changed it to -a/b.thanks

Aditya Anand - 6 years, 7 months ago

0 pending reports

×

Problem Loading...

Note Loading...

Set Loading...